Subordinate matrix norm inequality in research paper where authors replace $||A||_op^2$ with $||A^TA||_op$ The 2019 Stack Overflow Developer Survey Results Are InMatrix norm inequality implying eigenvector norm inequalityProperty of Subordinate Matrix Norm: $|AB| leq |A||B|$How can we replace $sup$ with $max$ in definition of subordinate norm for finite-dimensional vector space?Matrix and Vector Norm with diagonal matricesTriangle Inequality for SPD Matrix NormInequality between 2 norm and 1 norm of a matrixA matrix norm inequalitySubordinate matrix norm of 1-normInequality regarding norm of a positive definite matrixDeriving an inequality related to an induced matrix norm

Should I use my personal e-mail address, or my workplace one, when registering to external websites for work purposes?

What does "fetching by region is not available for SAM files" means?

Can a flute soloist sit?

Why did Acorn's A3000 have red function keys?

Are there any other methods to apply to solving simultaneous equations?

Which Sci-Fi work first showed weapon of galactic-scale mass destruction?

Identify This Plant (Flower)

Resizing object distorts it (Illustrator CC 2018)

How to answer pointed "are you quitting" questioning when I don't want them to suspect

What to do when moving next to a bird sanctuary with a loosely-domesticated cat?

Aging parents with no investments

What tool would a Roman-age civilization have for the breaking of silver and other metals into dust?

How to type this arrow in math mode?

Why isn't airport relocation done gradually?

Why isn't the circumferential light around the M87 black hole's event horizon symmetric?

If I score a critical hit on an 18 or higher, what are my chances of getting a critical hit if I roll 3d20?

What is the closest word meaning "respect for time / mindful"

Did Section 31 appear in Star Trek: The Next Generation?

Multiply Two Integer Polynomials

How are circuits which use complex ICs normally simulated?

Have you ever entered Singapore using a different passport or name?

If a Druid sees an animal’s corpse, can they wild shape into that animal?

Protecting Dualbooting Windows from dangerous code (like rm -rf)

FPGA - DIY Programming



Subordinate matrix norm inequality in research paper where authors replace $||A||_op^2$ with $||A^TA||_op$



The 2019 Stack Overflow Developer Survey Results Are InMatrix norm inequality implying eigenvector norm inequalityProperty of Subordinate Matrix Norm: $|AB| leq |A||B|$How can we replace $sup$ with $max$ in definition of subordinate norm for finite-dimensional vector space?Matrix and Vector Norm with diagonal matricesTriangle Inequality for SPD Matrix NormInequality between 2 norm and 1 norm of a matrixA matrix norm inequalitySubordinate matrix norm of 1-normInequality regarding norm of a positive definite matrixDeriving an inequality related to an induced matrix norm










1












$begingroup$


I'm perusing this paper. In page 8, I came across this:



enter image description here



My question is about equation 45. Also in this question I don't care about neither $u$ nor $M$, I mentioned them just to give some context.



In short my problem is:



Let $y in mathbbR^n$, $x in mathbbR^m$, $A in mathbbR^n times m$.



$|.|$ is the euclidean vector norm, $|.|_op$ is a matrix norm defined as:
$$|A|_op = max_x frac$$



So I have: $|y^TAx| leq |y|space|A|_op|x|$. As long as I have an inequality with the absolute value of something, it is the same as $y^TAx leq |y|space|A|_op|x|$



Now I will multiply both sides by 2 and multiply the left side by:
$$fracsqrttausqrttaufrac(tausigma (tausigma $$



Then I get:
$$2y^TAx leq 2
fracA
(tausigma
fracA^TA
sqrttau$$



using $a^2+b^2 geq 2ab$:
$$2y^TAx leq
frac^2_op tau
(tausigma |y|^2 +
fracA^TA
tau |x|^2 $$



In equation 45, the term $|A|^2_op$ doesn't appeard, it is replaced by $|A^TA|_op$. I know that $|A|^2_op geq |A^TA|_op$, so I don't know why the authors make this replacement. In the paper, they say that $A$ is any matrix with no special properties.



The next line comes from doing:
$$(tausigma |A^TA|_op)^frac12z = fracA^TA
(tausigma $$

$$tausigma |A^TA|_op z = |A^TA|_op tau$$
$$z = frac1sigma$$



Why do authors replace $|A|_op^2$ with $|A^TA|_op$ in equation 45? With this replacement the inequalities don't necessarily hold. Is there a property I am missing?










share|cite|improve this question











$endgroup$







  • 1




    $begingroup$
    Hint: Use | instead of || to get better-looking double bars ($|A|$ instead of $||A||$).
    $endgroup$
    – Rahul
    Mar 31 at 4:15











  • $begingroup$
    @Rahul thanks for the hint, I didn't know about it. I'll use it next time.
    $endgroup$
    – Broken_Window
    Apr 2 at 21:15















1












$begingroup$


I'm perusing this paper. In page 8, I came across this:



enter image description here



My question is about equation 45. Also in this question I don't care about neither $u$ nor $M$, I mentioned them just to give some context.



In short my problem is:



Let $y in mathbbR^n$, $x in mathbbR^m$, $A in mathbbR^n times m$.



$|.|$ is the euclidean vector norm, $|.|_op$ is a matrix norm defined as:
$$|A|_op = max_x frac$$



So I have: $|y^TAx| leq |y|space|A|_op|x|$. As long as I have an inequality with the absolute value of something, it is the same as $y^TAx leq |y|space|A|_op|x|$



Now I will multiply both sides by 2 and multiply the left side by:
$$fracsqrttausqrttaufrac(tausigma (tausigma $$



Then I get:
$$2y^TAx leq 2
fracA
(tausigma
fracA^TA
sqrttau$$



using $a^2+b^2 geq 2ab$:
$$2y^TAx leq
frac^2_op tau
(tausigma |y|^2 +
fracA^TA
tau |x|^2 $$



In equation 45, the term $|A|^2_op$ doesn't appeard, it is replaced by $|A^TA|_op$. I know that $|A|^2_op geq |A^TA|_op$, so I don't know why the authors make this replacement. In the paper, they say that $A$ is any matrix with no special properties.



The next line comes from doing:
$$(tausigma |A^TA|_op)^frac12z = fracA^TA
(tausigma $$

$$tausigma |A^TA|_op z = |A^TA|_op tau$$
$$z = frac1sigma$$



Why do authors replace $|A|_op^2$ with $|A^TA|_op$ in equation 45? With this replacement the inequalities don't necessarily hold. Is there a property I am missing?










share|cite|improve this question











$endgroup$







  • 1




    $begingroup$
    Hint: Use | instead of || to get better-looking double bars ($|A|$ instead of $||A||$).
    $endgroup$
    – Rahul
    Mar 31 at 4:15











  • $begingroup$
    @Rahul thanks for the hint, I didn't know about it. I'll use it next time.
    $endgroup$
    – Broken_Window
    Apr 2 at 21:15













1












1








1





$begingroup$


I'm perusing this paper. In page 8, I came across this:



enter image description here



My question is about equation 45. Also in this question I don't care about neither $u$ nor $M$, I mentioned them just to give some context.



In short my problem is:



Let $y in mathbbR^n$, $x in mathbbR^m$, $A in mathbbR^n times m$.



$|.|$ is the euclidean vector norm, $|.|_op$ is a matrix norm defined as:
$$|A|_op = max_x frac$$



So I have: $|y^TAx| leq |y|space|A|_op|x|$. As long as I have an inequality with the absolute value of something, it is the same as $y^TAx leq |y|space|A|_op|x|$



Now I will multiply both sides by 2 and multiply the left side by:
$$fracsqrttausqrttaufrac(tausigma (tausigma $$



Then I get:
$$2y^TAx leq 2
fracA
(tausigma
fracA^TA
sqrttau$$



using $a^2+b^2 geq 2ab$:
$$2y^TAx leq
frac^2_op tau
(tausigma |y|^2 +
fracA^TA
tau |x|^2 $$



In equation 45, the term $|A|^2_op$ doesn't appeard, it is replaced by $|A^TA|_op$. I know that $|A|^2_op geq |A^TA|_op$, so I don't know why the authors make this replacement. In the paper, they say that $A$ is any matrix with no special properties.



The next line comes from doing:
$$(tausigma |A^TA|_op)^frac12z = fracA^TA
(tausigma $$

$$tausigma |A^TA|_op z = |A^TA|_op tau$$
$$z = frac1sigma$$



Why do authors replace $|A|_op^2$ with $|A^TA|_op$ in equation 45? With this replacement the inequalities don't necessarily hold. Is there a property I am missing?










share|cite|improve this question











$endgroup$




I'm perusing this paper. In page 8, I came across this:



enter image description here



My question is about equation 45. Also in this question I don't care about neither $u$ nor $M$, I mentioned them just to give some context.



In short my problem is:



Let $y in mathbbR^n$, $x in mathbbR^m$, $A in mathbbR^n times m$.



$|.|$ is the euclidean vector norm, $|.|_op$ is a matrix norm defined as:
$$|A|_op = max_x frac$$



So I have: $|y^TAx| leq |y|space|A|_op|x|$. As long as I have an inequality with the absolute value of something, it is the same as $y^TAx leq |y|space|A|_op|x|$



Now I will multiply both sides by 2 and multiply the left side by:
$$fracsqrttausqrttaufrac(tausigma (tausigma $$



Then I get:
$$2y^TAx leq 2
fracA
(tausigma
fracA^TA
sqrttau$$



using $a^2+b^2 geq 2ab$:
$$2y^TAx leq
frac^2_op tau
(tausigma |y|^2 +
fracA^TA
tau |x|^2 $$



In equation 45, the term $|A|^2_op$ doesn't appeard, it is replaced by $|A^TA|_op$. I know that $|A|^2_op geq |A^TA|_op$, so I don't know why the authors make this replacement. In the paper, they say that $A$ is any matrix with no special properties.



The next line comes from doing:
$$(tausigma |A^TA|_op)^frac12z = fracA^TA
(tausigma $$

$$tausigma |A^TA|_op z = |A^TA|_op tau$$
$$z = frac1sigma$$



Why do authors replace $|A|_op^2$ with $|A^TA|_op$ in equation 45? With this replacement the inequalities don't necessarily hold. Is there a property I am missing?







matrices norm matrix-norms






share|cite|improve this question















share|cite|improve this question













share|cite|improve this question




share|cite|improve this question








edited Apr 2 at 21:34









Martin Argerami

129k1184185




129k1184185










asked Mar 30 at 16:48









Broken_WindowBroken_Window

267211




267211







  • 1




    $begingroup$
    Hint: Use | instead of || to get better-looking double bars ($|A|$ instead of $||A||$).
    $endgroup$
    – Rahul
    Mar 31 at 4:15











  • $begingroup$
    @Rahul thanks for the hint, I didn't know about it. I'll use it next time.
    $endgroup$
    – Broken_Window
    Apr 2 at 21:15












  • 1




    $begingroup$
    Hint: Use | instead of || to get better-looking double bars ($|A|$ instead of $||A||$).
    $endgroup$
    – Rahul
    Mar 31 at 4:15











  • $begingroup$
    @Rahul thanks for the hint, I didn't know about it. I'll use it next time.
    $endgroup$
    – Broken_Window
    Apr 2 at 21:15







1




1




$begingroup$
Hint: Use | instead of || to get better-looking double bars ($|A|$ instead of $||A||$).
$endgroup$
– Rahul
Mar 31 at 4:15





$begingroup$
Hint: Use | instead of || to get better-looking double bars ($|A|$ instead of $||A||$).
$endgroup$
– Rahul
Mar 31 at 4:15













$begingroup$
@Rahul thanks for the hint, I didn't know about it. I'll use it next time.
$endgroup$
– Broken_Window
Apr 2 at 21:15




$begingroup$
@Rahul thanks for the hint, I didn't know about it. I'll use it next time.
$endgroup$
– Broken_Window
Apr 2 at 21:15










1 Answer
1






active

oldest

votes


















2












$begingroup$

I'll use $langle x,yrangle$ to denote the usual inner product $y^*x$.



It is always true that $$|A|^2_op=|A^*A|_op,$$ where $A^*$ denotes the adjoint (the conjugate transpose, so just the transpose if $A$ is real). This is very easy to prove. The reverse inequality from the one you mention is the easy one:
beginalign
|A|^2&=maxAx=maxx\
&=maxx\
&leq |A^*A|,
endalign

where the inequality is simply Cauchy-Schwarz and definition of the operator norm. Now, from the above inequality, you have
$$
|A|^2leq|A^*A|leq|A^*|,|A|.
$$

So, for nonzero $A$ (the zero case is trivial) you have $|A|leq|A^*|$. Apply the above to $A^*$, to get $|A^*|leq|A^**|=|A|$, so $|A^*|=|A|$. If you now go back to the inequalities we had,
$$
|A|^2leq|A^*A|leq|A^*|,|A|=|A|^2.
$$

So $|A|^2=|A^*A|$.






share|cite|improve this answer









$endgroup$













    Your Answer





    StackExchange.ifUsing("editor", function ()
    return StackExchange.using("mathjaxEditing", function ()
    StackExchange.MarkdownEditor.creationCallbacks.add(function (editor, postfix)
    StackExchange.mathjaxEditing.prepareWmdForMathJax(editor, postfix, [["$", "$"], ["\\(","\\)"]]);
    );
    );
    , "mathjax-editing");

    StackExchange.ready(function()
    var channelOptions =
    tags: "".split(" "),
    id: "69"
    ;
    initTagRenderer("".split(" "), "".split(" "), channelOptions);

    StackExchange.using("externalEditor", function()
    // Have to fire editor after snippets, if snippets enabled
    if (StackExchange.settings.snippets.snippetsEnabled)
    StackExchange.using("snippets", function()
    createEditor();
    );

    else
    createEditor();

    );

    function createEditor()
    StackExchange.prepareEditor(
    heartbeatType: 'answer',
    autoActivateHeartbeat: false,
    convertImagesToLinks: true,
    noModals: true,
    showLowRepImageUploadWarning: true,
    reputationToPostImages: 10,
    bindNavPrevention: true,
    postfix: "",
    imageUploader:
    brandingHtml: "Powered by u003ca class="icon-imgur-white" href="https://imgur.com/"u003eu003c/au003e",
    contentPolicyHtml: "User contributions licensed under u003ca href="https://creativecommons.org/licenses/by-sa/3.0/"u003ecc by-sa 3.0 with attribution requiredu003c/au003e u003ca href="https://stackoverflow.com/legal/content-policy"u003e(content policy)u003c/au003e",
    allowUrls: true
    ,
    noCode: true, onDemand: true,
    discardSelector: ".discard-answer"
    ,immediatelyShowMarkdownHelp:true
    );



    );













    draft saved

    draft discarded


















    StackExchange.ready(
    function ()
    StackExchange.openid.initPostLogin('.new-post-login', 'https%3a%2f%2fmath.stackexchange.com%2fquestions%2f3168510%2fsubordinate-matrix-norm-inequality-in-research-paper-where-authors-replace-a%23new-answer', 'question_page');

    );

    Post as a guest















    Required, but never shown

























    1 Answer
    1






    active

    oldest

    votes








    1 Answer
    1






    active

    oldest

    votes









    active

    oldest

    votes






    active

    oldest

    votes









    2












    $begingroup$

    I'll use $langle x,yrangle$ to denote the usual inner product $y^*x$.



    It is always true that $$|A|^2_op=|A^*A|_op,$$ where $A^*$ denotes the adjoint (the conjugate transpose, so just the transpose if $A$ is real). This is very easy to prove. The reverse inequality from the one you mention is the easy one:
    beginalign
    |A|^2&=maxAx=maxx\
    &=maxx\
    &leq |A^*A|,
    endalign

    where the inequality is simply Cauchy-Schwarz and definition of the operator norm. Now, from the above inequality, you have
    $$
    |A|^2leq|A^*A|leq|A^*|,|A|.
    $$

    So, for nonzero $A$ (the zero case is trivial) you have $|A|leq|A^*|$. Apply the above to $A^*$, to get $|A^*|leq|A^**|=|A|$, so $|A^*|=|A|$. If you now go back to the inequalities we had,
    $$
    |A|^2leq|A^*A|leq|A^*|,|A|=|A|^2.
    $$

    So $|A|^2=|A^*A|$.






    share|cite|improve this answer









    $endgroup$

















      2












      $begingroup$

      I'll use $langle x,yrangle$ to denote the usual inner product $y^*x$.



      It is always true that $$|A|^2_op=|A^*A|_op,$$ where $A^*$ denotes the adjoint (the conjugate transpose, so just the transpose if $A$ is real). This is very easy to prove. The reverse inequality from the one you mention is the easy one:
      beginalign
      |A|^2&=maxAx=maxx\
      &=maxx\
      &leq |A^*A|,
      endalign

      where the inequality is simply Cauchy-Schwarz and definition of the operator norm. Now, from the above inequality, you have
      $$
      |A|^2leq|A^*A|leq|A^*|,|A|.
      $$

      So, for nonzero $A$ (the zero case is trivial) you have $|A|leq|A^*|$. Apply the above to $A^*$, to get $|A^*|leq|A^**|=|A|$, so $|A^*|=|A|$. If you now go back to the inequalities we had,
      $$
      |A|^2leq|A^*A|leq|A^*|,|A|=|A|^2.
      $$

      So $|A|^2=|A^*A|$.






      share|cite|improve this answer









      $endgroup$















        2












        2








        2





        $begingroup$

        I'll use $langle x,yrangle$ to denote the usual inner product $y^*x$.



        It is always true that $$|A|^2_op=|A^*A|_op,$$ where $A^*$ denotes the adjoint (the conjugate transpose, so just the transpose if $A$ is real). This is very easy to prove. The reverse inequality from the one you mention is the easy one:
        beginalign
        |A|^2&=maxAx=maxx\
        &=maxx\
        &leq |A^*A|,
        endalign

        where the inequality is simply Cauchy-Schwarz and definition of the operator norm. Now, from the above inequality, you have
        $$
        |A|^2leq|A^*A|leq|A^*|,|A|.
        $$

        So, for nonzero $A$ (the zero case is trivial) you have $|A|leq|A^*|$. Apply the above to $A^*$, to get $|A^*|leq|A^**|=|A|$, so $|A^*|=|A|$. If you now go back to the inequalities we had,
        $$
        |A|^2leq|A^*A|leq|A^*|,|A|=|A|^2.
        $$

        So $|A|^2=|A^*A|$.






        share|cite|improve this answer









        $endgroup$



        I'll use $langle x,yrangle$ to denote the usual inner product $y^*x$.



        It is always true that $$|A|^2_op=|A^*A|_op,$$ where $A^*$ denotes the adjoint (the conjugate transpose, so just the transpose if $A$ is real). This is very easy to prove. The reverse inequality from the one you mention is the easy one:
        beginalign
        |A|^2&=maxAx=maxx\
        &=maxx\
        &leq |A^*A|,
        endalign

        where the inequality is simply Cauchy-Schwarz and definition of the operator norm. Now, from the above inequality, you have
        $$
        |A|^2leq|A^*A|leq|A^*|,|A|.
        $$

        So, for nonzero $A$ (the zero case is trivial) you have $|A|leq|A^*|$. Apply the above to $A^*$, to get $|A^*|leq|A^**|=|A|$, so $|A^*|=|A|$. If you now go back to the inequalities we had,
        $$
        |A|^2leq|A^*A|leq|A^*|,|A|=|A|^2.
        $$

        So $|A|^2=|A^*A|$.







        share|cite|improve this answer












        share|cite|improve this answer



        share|cite|improve this answer










        answered Mar 31 at 4:02









        Martin ArgeramiMartin Argerami

        129k1184185




        129k1184185



























            draft saved

            draft discarded
















































            Thanks for contributing an answer to Mathematics Stack Exchange!


            • Please be sure to answer the question. Provide details and share your research!

            But avoid


            • Asking for help, clarification, or responding to other answers.

            • Making statements based on opinion; back them up with references or personal experience.

            Use MathJax to format equations. MathJax reference.


            To learn more, see our tips on writing great answers.




            draft saved


            draft discarded














            StackExchange.ready(
            function ()
            StackExchange.openid.initPostLogin('.new-post-login', 'https%3a%2f%2fmath.stackexchange.com%2fquestions%2f3168510%2fsubordinate-matrix-norm-inequality-in-research-paper-where-authors-replace-a%23new-answer', 'question_page');

            );

            Post as a guest















            Required, but never shown





















































            Required, but never shown














            Required, but never shown












            Required, but never shown







            Required, but never shown

































            Required, but never shown














            Required, but never shown












            Required, but never shown







            Required, but never shown







            Popular posts from this blog

            Triangular numbers and gcdProving sum of a set is $0 pmod n$ if $n$ is odd, or $fracn2 pmod n$ if $n$ is even?Is greatest common divisor of two numbers really their smallest linear combination?GCD, LCM RelationshipProve a set of nonnegative integers with greatest common divisor 1 and closed under addition has all but finite many nonnegative integers.all pairs of a and b in an equation containing gcdTriangular Numbers Modulo $k$ - Hit All Values?Understanding the Existence and Uniqueness of the GCDGCD and LCM with logical symbolsThe greatest common divisor of two positive integers less than 100 is equal to 3. Their least common multiple is twelve times one of the integers.Suppose that for all integers $x$, $x|a$ and $x|b$ if and only if $x|c$. Then $c = gcd(a,b)$Which is the gcd of 2 numbers which are multiplied and the result is 600000?

            Ingelân Ynhâld Etymology | Geografy | Skiednis | Polityk en bestjoer | Ekonomy | Demografy | Kultuer | Klimaat | Sjoch ek | Keppelings om utens | Boarnen, noaten en referinsjes Navigaasjemenuwww.gov.ukOffisjele webside fan it regear fan it Feriene KeninkrykOffisjele webside fan it Britske FerkearsburoNederlânsktalige ynformaasje fan it Britske FerkearsburoOffisjele webside fan English Heritage, de organisaasje dy't him ynset foar it behâld fan it Ingelske kultuergoedYnwennertallen fan alle Britske stêden út 'e folkstelling fan 2011Notes en References, op dizze sideEngland

            Boston (Lincolnshire) Stedsbyld | Berne yn Boston | NavigaasjemenuBoston Borough CouncilBoston, Lincolnshire